Electromagnetism – Magnetic Force Between Two Parallel Moving Charged Particles

electromagnetismspecial-relativity

From what I've read on magnetism as a result of incorporating special relativity into electrostatics, I have got the impression that:

a) If you have two parallel and (for simplicity) infinite streams of moving particles with equal velocity, in the particles' frame they observe each other to be stationary. The Coulomb force on any one particle is given by $qE$ where E is resultant field.

b) Viewing the same situation from a laboratory reference frame, we observe a length contraction for both streams. Then the particles appear closer to each other, so the density appears to us to be more than what the particle observes. So we predict a larger Coulomb force of repulsion than the actual force that the particle feels, and introduce the magnetic field as a 'correction'.
For a section of aforementioned infinite streams of length x and separation r:
$$
F_{net} = (F_{coul})_{frame} = (F_{coul})_{obs} + F_b
\implies F_b = (F_{coul})_{frame} – (F_{coul})_{obs}
$$

$$
F_b = \frac{1}{2\pi\epsilon_0r}\lambda_{frame}^2x – \frac{1}{2\pi\epsilon_0r}\lambda_{obs}^2x
$$

Now by apparent length contraction of the particle stream in the lab frame: $\lambda_{frame} = \lambda_{obs}\sqrt{1 – v^2/c^2} $
$$
\implies F_b = -\frac{1}{2\pi\epsilon_0r}\lambda_{obs}^2x(1-\frac{c^2 – v^2}{c^2}) = -\frac{\lambda_{obs}^2v^2x}{2\pi\epsilon_0rc^2} = -\frac{i^2x}{2\pi\epsilon_0rc^2} =-\mu_0\frac{i^2x}{2\pi r}
$$

I.e. the force is attractive in nature and this agrees with $ c^2 = \frac{1}{\mu_0\epsilon_0} $ and what was predicted by Biot-Savart's law.

The question arises what happens if there is no stream but just, say, two parallel particles moving with the same velocity? There is no question of any charge density observed or otherwise. But in the particle's frame of reference there is just the Coulomb force, and in our frame of reference there is also the Coulomb force, which should not change as there is no length contraction observed. What am I getting wrong here? Surely one moving charged particle generates a magnetic field (which I was told is true in any frame where the charge moves), and the field would act on the parallel particle? How do we explain this with special relativity?

Best Answer

So imagine two point, positive charges at rest, in a frame S: the interaction is purely Coulombic, and the force on one another has magnitude $\frac{q}{4\pi\epsilon_0r^2}$, $r$ is the distance between them. Now consider the events from a frame, which I denote as $\bar S$, moving along the x-axis at velocity v: the charges are moving according to this frame. Now, the transformation for fields in special relativity is as follows: suppose one has a frame $S$ with electric and magnetic fields denoted by $(E_x, E_y, E_z)$ and $(B_x, B_y, B_z)$ and another frame $\bar S$ moving at speed v, along the x-axis, relative to S, where the fields are given by $(\bar{E_x},\bar{E_y},\bar{E_z})$ and $(\bar{B_x},\bar{B_y},\bar{B_z})$. Then,

$\bar{E_x} = E_x, \bar{B_x}=B_x$

$\bar{E_y} = \gamma(E_y-vB_z), \bar{B_z}=\gamma(B_z-\frac{v}{c^2}E_y)$

$\bar{E_z} = \gamma(E_z+vB_y), \bar{B_y}=\gamma(B_y+\frac{v}{c^2}E_z)$

This derivation can be found in textbooks like Griffiths' Introduction to Electrodynamics or Feynman's Lectures. In our case, from $S$ there is only an electric field, which will have three components, and a magnetic field which is zero. However, when looking from $\bar S$, there are electric as well as magnetic fields due to the transformation equation I have stated. So yeah, the Coulombic field does change, and we also have a magnetic field here!

Edit: Okay, I am so sorry, I understood your question now, and I'm removing the previous edit. As someone else has remarked here, there is a transformation law for the perpendicular force acting on the particle, which goes as $\bar F = \frac{F}{\gamma}.$ Here, what's important is that from one of the frames, the particle must be at rest, and F is the force on the particle as measured from this frame and $\bar F$ in the other frame. Now for your case, imagine two charges moving at some velocity $\bar u$ along the x-axis relative to a frame $\bar S$, one above the other. $\bar S$ itself is moving at velocity v along the x-axis relative to another frame $S$. What's special about S is that the charges are at rest from this frame(This translates to $\bar u = -v$, but I'll leave you to worry about how that happens!). So, the force between the charges is purely electrostatic, and I will call the direction of the force along the y-axis, and $F=\frac{q^2}{4\pi\epsilon_0y^2}$, say, on the upper charge. Now, from our rule, $\bar F = \frac{F}{\gamma}$, we find that in the frame $\bar S$, the force is reduced. However, we know that by applying relativity to a single charge, the electric field from $\bar S$ should be $\gamma E$, where $E$ is the field from $S$. so we expect the force to also increase by $\gamma$, which does not happen. This hints at the presence of another frame-dependent force acting opposite to the electric field in this frame, and it's of course what we call a 'magnetic force!' Hope this helped!

Related Question